Sei sulla pagina 1di 45

PROBLEM 34. CE Board May 2009 540 106 = 0.90(300)2 (28)(0.

21)
A simply reinforced concrete beam reinforced for = 583.21
tension has a width of 300 mm and a total depth
of 600 mm. It is subjected to an external moment
Mu = 540 kN-m, fc = 28 MPa, fy = 280 MPa, Es
= 200 GPa.
Which of the following gives the balance steel
ration in percent.
Which of the following gives the depth a in
terms of d in percent using = b.
Which of the following gives the minimum
effective depth.

= ( /2)
Solution:
= 0.85 ( /2)
Balanced steel ratio:
540 106 = 0.85(28)()(300)(583.21 /
0.85 600 2)
= (600+ )

75630.25 = (583.21 0.5)
0.85(25)(0.85)
=
280 (600+ 280) 0.52 583.21 + 75630.25 = 0
= 0.04926% 2 1166.42 + 151260.50 = 0
= . % 1166.42869.19
= 2

= 148.61
Depth a in terms of d in percent: 148.61
=
2 583.21
= (1 0.59)

1
= 0.2548
= 2

1
= . %
= (0.04926)
2

= 0.0246
Min. effective depth:

= .

=

0.0246(280)
= 28
= 0.246 PROBLEM 35. CE Board Nov. 2009

(1 0.59) = 0.246(1 0.59 0.246) A rectangular beam has a width of 300 mm and
an effective depth of 460 mm. The beam is
= 2 (1 0.59) reinforced with 2-28 mm at the top.
fc = 35 MPa, fy = 350 MPa. = . %
Compute the ratio of the depth of compression
block to the distance of the top fiber to the neutral
axis. Max. area of steel permitted:

Compute the balanced steel ratio of the =


reinforcement. = 0.75
Compute the max. area of steel permitted. = 0.75(0.0429)(300)(460)

= .
Solution:
Ratio of the depth of compression block to the PROBLEM 36:
distance of the top fiber to the neutral axis
A rectangular beam having a width of 300 mm
and an effective depth of 450 mm. It is reinforced
with 4-36 mm in diameter bars.
fc = 28 MPa, fy = 270 MPa, Es = 200000
MPa.
Compute the depth of compression block for
a balanced condition.
Compute the nominal moment capacity of the
beam.
If the value of fc is increased by 25%,
compute the percentage of the increased nominal
a = depth of compression block
moment capacity of the beam.
c = distance of top fiber to neutral axis
=
Solution:

= Depth of compression block for a balanced
condition:
0.05( 28)
= 0.85
7

0.05(3528)
= 0.85 7

= .

Balanced steel ratio of the reinforcement:


0.85 600
=
(600+ )
0.003 0.00135
0.85(35)(0.80)(600)
= 450
= 350 (600+ 350)
1.35 0.003 = 0.00135 Percentage of the increased nominal moment
capacity of the beam if fc is increased by 25%:
0.00435 = 1.35

= 4 (36)2 (4)
= 310.34
= = 1296

= ( 2 )
Note:
= 0.85 = 28 =
= 0.85(310.34) 0.85 =
= . = 1.28(28)
= 35
Nominal moment capacity:

0.85 =
0.85(35)()(300) = 1296(270)
= 123.17


= ( 2 )

123.17
= 1296(270) (450 2
)

= 427 106
= = 427

0.85 =

0.85(28)()(300) = 4 (36)2 (4)(270) Percentage increase in nominal moment:
427410
= 153.96 % = ( ) 100
410

% = . %

= ( 2 )

153.96
= 4 (36)2 (4)(270) (450 2
) PROBLEM 37:

= 410 106 A reinforced concrete beam has a width of 400


mm and an effective depth of 600 mm. It is
= reinforced for tension with 4-28 mm bars. fc =
20.7 MPa, fy = 414.6 MPa.
Determine the percent increase in nominal = 0.00207
moment if the depth is increased to 700 mm.
> ( )
Determine the percent increase in nominal
moment if fc is increased to 27.6 MPa.
Determine the percent increase in nominal Nominal moment if d = 600 mm:
moment if the steel is changed to 4-32 mm .
= ( 2 )

145.09
= 0.90 (28)2 (4)(414.6) (600 )
Solution: 4 2

Percent increase in nominal moment if the = 484755959


depth is increased to 700 mm: = 484756

Nominal moment if d = 700 mm:



= ( 2 )

145.09
= 0.90 (28)2 (4)(414.6) (700 )
4 2

= 576660663

= = 576661

0.85 =
Percent increase in nominal moment:
0.85(20.7)()(400) = 4 (28)2 (4)(414.6)
576661484756
= 145.09 % = ( ) 100
484756

% = . %
=
145.09 = 0.85 Percent increase in nominal moment if fc is
= 170.69 increased to 27.6 MPa:
=
0.003 0.85 =
429.31
= 170.69

0.85(27.6)()(400) = 4 (28)2 (4)(414.6)
= 0.00754
= 108.82


=


= ( 2 )
414.6
= 200,000
108.82 Determine the moment capacity using
= 0.90 (28)2 (4)(414.6) (600 )
4 2
moment reduction factor of 0.90.
= 501423048.1
Determine the super-imposed uniform live
= 501423 load it could carry in kPa besides a dead load of
20 kN/m including its own weight if it has a
simple span of 6 m and a spacing of 1.8m.
Percentage increase in nominal moment:
501423484756
% = ( ) 100 Solution:
484756

Depth of compression block:


% = . %

Percent increase in nominal moment if the


steel is change to 4-32 mm :

=
0.85 =

0.85(20.7)()(400) = (32)2 (4)(414.6)
4

= 4 (32)2 (3)
= 189.51

= 2413 2

= ( 2 )
=
189.51
= 0.90 (32)2 (4)(414.6) (600 ) 2413
4 2
= 300(575)
= 606490
= 0.014

606490484756
% = ( ) 100
484756

= 0.75
% = . %
= 0.75(0.02850
PROBLEM 38:
= 0.021 > 0.014
A reinforced concrete beam has a width of 300
mm with an effective depth of 575 mm. It is : =
reinforced with 3-32 mm at the bottom. fc =
27.6 MPa, fy = 414 MPa. Balanced steel ratio b
= 0.0285.
=
Determine the depth of compression block.
0.85 =

0.85(27.6)()(300) = 4 (2413)(414) = 47.98

= .
47.98
= 1.8
Moment capacity using moment reduction
= .
factor of 0.90:
=
PROBLEM 39:
141.44 = 0.85
A fixed ended rectangular beam must support a
= 166.99
uniform service dead and live loads of 220 kN/m
and 182.6 kN/m respectively. It has a span of 6m.
fc = 27.6 MPa, fy = 414.7 MPa, b = 0.028.
Check whether we could use = 0.90:
Effective depth of the beam using = 0.18.
1 0.003
= Compute the flexural reinforcements at the
408.01 166.99
support.
1 = 0.0073 > 0.005
Compute for the flexural reinforcements at the
= 0.90 midspan.


= ( ) Solution:
2

= 0.90 (2413)(414) (575


141.44
) Effective depth of the beam using = 0.18:
2

= 453.4

Super-imposed uniform live load it could carry


in kPa besides a dead load of 20 kN/m including
its own weight if it has a simple span of 6 m and
a spacing of 1.8m:
= 1.2 + 1.6
2
= 8 = 1.2(220) + 1.6(182.6)
(6)2
453.4 = 8
= 556.2 /

= 100.76
Max. moment occurs at the fixed supports:
2
= 1.6 + 1.2 =
12

556.2(6)2
100.76 = 1.6 + 1.2(20) = 12
= 1668.60 <

Moment at midspan:
2 =
+ =
24

556.2(6)2
= 0.01198(600)(850)
+ = 24
=
+ = 834.3
Flexural reinforcements at the midspan:

+ = 834.3
Effective depth of beam:
= 2 (1 0.59)
= 2 (1 0.59)
843.3 106 =
6 2
1668.6 10 = 0.90(27.6)(600) (0.18)[1 0.90(27.6)(600)(850)2 ()[1 0.59()]
0.59(0.18)]
[1 0.59] = 0.0774
= 834.2 850
2 1.6949 + 0.1313 = 0
=
= 0.0814

Flexural reinforcements at the supports:



=
= 0.18

0.0814(27.6)
=
=
414.7

0.18(27.6) = 0.00541
= 414.7

= 0.01198 14
= 414.7

14 = 0.00338
= 414.7

=
<
0.00338
0.00541 < 0.021

=
0.75
= 0.75(0.028) =
= 0.01198 = 0.00541(600)(850)
= 0.021 =
PROBLEM 40:
A rectangular concrete beam has a width of 250
mm and a total depth of 450 mm. It is reinforced
with a total steel area of 1875 mm2 placed at an
effective depth of 375 mm. fc = 27.6 MPa, fy =
414.7 MPa.
Determine the depth of compression block. 0.003
219.02
= 155.98
Determine the moment capacity reduction
factor. = 0.0042125 > 0.002 < 0.005

Determine the safe live load that the beam


could carry in addition to a dead load of 20 kN/m
=
if it has a span of 6m.

414.7
= 200,000
Solution: = 0.0020735 < 0.0042125
Depth of compression block:
Steel yields:

Assuming steel yields:


=
Since t is between 0.002 and 0.005, this value is
0.85 = within the transition range between compression
controlled section and tension controlled section.
0.85(27.6)()(250) = 1875(414.7)
250
= . = 0.65 + ( 0.002) 3
250
= 0.65 + (0.0042125 0.002) 3
Moment capacity reduction factor:
= .
=
132.58 = 0.85
Live load it could carry:
= 155.98
= ( 2 )
132.58 Wu = 1.2 (10.5) + 1.6 (22)
= 0.834 (1875)(414.7) (375 2
)
Wu = 47.8 kN/m
= 200.2 10 6
2
= 8
= 200.2
47.8 (6)2
= 8

2 = . .
= 8

(6)2
200.2 = 8
2. Approximate flexural resistance factor:

= 44.48 / Mu = R b d2
d = 400 62.5 = 337.50
215.1 x 106 = 0.90 R (300) (337.5)2
= 1.2 + 1.6
R = 6.99
44.48 = 1.2(20) + 1.6

= .
3. Number of 32 mm bars:
R = fc (1 0.59 )
6.99 = 27.6 (1 0.59 )
PROBLEM 41:
0.59 2 27.6 + 6.99 = 0
Architectural considerations limit the height of a
2 46.78 + 11.85 = 0
6 m. long simple span beam to 400 mm and width
of 300 mm. The following loads are material
46.78 46.27
properties are given: Use 62.5 mm as covering = 2
= 0.25

from center of reinforcing bars.


=

DL = 10.5 kN/m LL = 22 kN/m (414.7)
0.25 =
fc = 34.6 MPa fy = 414.7 MPa 34.6

= 0.0209

1. Determine the factored moment carried


by the beam. As = b d

2. Determine the approximate flexural As = 0.0209 (300) (337.50)

resistance factor. Assume = 0.90 As = 2112 mm2

3. Determine the number of 32 mm bars



needed for the beam. = (32)2 = 2112
4

= 2.63 3
Solution: Use 3 32 mm bars
1. Factored moment carried by the beam.
As = 4 = (32)2 (3) = 2413 2
Wu = 1.2 DL + 1.6 LL
= 0.75
0.85 600 (250)
= Use = 0.65 + ( 0.002) 3
(600+ )
(250)
0.05 ( 30) = 0.65 + (0.00426 0.002)
= 0.85 7
3

0.05 (34.630) = 0.838 (moment capacity reduction factor)


= 0.85 7

= 0.817
Mu = As fy (d - 2)
0.85 (34.6)(0.817)(600)
= 414.7 (600+414.7) 113.42
Mu = 0.838 (2413) (414.7) (337.50 - 2
)
= 0.03426
Mu = 235.46 kN.m > 215.10 kN.m (safe)
= 0.75
= 0.75(0.03426)
PROBLEM 42:
= 0.0257
A reinforced rectangular concrete beam has a
width of 250 mm and an effective depth of 360

= mm. It is reinforced for tension only at the bottom
2413
= with a total tension steel area of 600 mm2. fc = 40
300(337.5)
MPa, fy = 400 MPa.
= 0.0238 < 0.0257 ok

1. Determine the tension reinforcement


index for this beam.
2. Determine the distance of the neutral axis
below the compression surface.
3. Determine the ultimate flexural strength
of the beam.

Solution:
C=T
1. Tension reinforcement index for this
0.85 fc ab = As fy
beam.
0.85 (34.6) (a) (300) = 2413 (414.7)
a = 113.42
a = c
113.42 = 0.817 c
c = 138.82
0.003
198.68
= 138.82

= 0.00426 < 0.005



=


=
600
= 250(360)

= 0.0067

=

0.0067 (400)
=
40

= . Mu = As fy (d - 2)
0.003
323.80
= 36.20
2. Distance of the neutral axis below the
= 0.0026983
compression surface.
0.05 ( 30)
= 0.85
7
=
0.05 (4030)
= 0.85 7 400
=
200000
= 0.78
= 0.002

Assume steel yields:


> steel yields (under reinforced)
C=T
0.85 fc ab = As fy =

0.85 (40) (a) (250) = 600 (400) = 0.0026983 < 0.005


a = 28.23 mm = moment capacity reduction factor
(250)
a = c = 0.70 + ( 0.002) 3
28.23 = 0.78 c
c = 36.20 mm (neutral axis
below compression surface)
3. Ultimate flexural strength of the beam.

(250)
Use = 0.65 + ( 0.002)
3
= 0.65 + (0.0026983 0.002)
(250) Steel does not yield: fs fy
3
Using Hookes Law, solve for the actual fs:
= 0.71
fs =
0.003
Mu = As fy (d - )
=
500
2
(500)(0.003)
Mu = 0.71 (600) (400) (360 -
28.23
) =

2
200000(500)(0.003)
Mu = 58.93 kN.m =

600(500)
=
PROBLEM 43:
A rectangular beam has a width of 280 mm and
T=C
an effective depth of 500 mm. It is reinforced with
As fy = 0.85 fc ab
4 36 mm bars at the tension side of the beam
(500)
4072 (600 ) = 0.85(25)(0.85)(280)
placed 65 mm above the bottom of the beam.

500 c = 0.00207 c2
1. Which of the following will give the c2 + 483.09 c 241546 = 0
location of the neutral axis from the top c = 306.08 mm
of the beam.
2. Which of the following give the stress of 2. Stress of steel.
steel. a = c
3. Which of the following will give the a = 0.85 (306.08)
ultimate capacity of the beam. a = 260.17 mm

Solution:
600 (500)
1. Location of the neutral axis from the top =
of the beam . 600 (500306.08)
= 306.08

= . <

3. Ultimate moment capacity of the beam



Mu = As fy (d - 2)
260.17
Mu = 0.90 (4072) (380.14) (500 - 2
)

Mu = 515.3 kN.m
(500)(0.003)
=
PROBLEM 44:
=
A rectangular beam has a width of 280 mm and

(500)(0.003)
an effective depth of 500 mm and is reinforced 200000
=
with steel area in tension equal to 4000 mm2. fc 200000(500)(0.003)
=
= 25 MPa, fy = 400 MPa.
600(500)
=
1. Compute the depth of compression strees

block.
2. Compute the ultimate moment capacity C=T
of the beam. 0.85 fc ab = As fs
3. What is the correct description of the 0.85 fc cb = 40000 fs
400(600)(500)
beam? 0.85 (25) (0.85) c (280) =
a. Under reinforced c2 + 474.54c 237271.4 = 0
b. Over reinforced c = 304.55
c. Balanced condition
d. Reduction in depth of 600(500)
=
compression zone results in
600(500304.55)
decrease in steel strain at failure. =
304.55

= 385.06 < = 400


Solution: Steel does not yield
1. Depth of compression stress block ok as assumed
Assume steel does not yield
a = c
a = 0.85 (304.55)
a = 258.87 mm depth of
compression block

2. Ultimate moment capacity of the beam



Mu = As fy (d - )
2
258.87
Mu = 0.90 (4000) (385.06) (500 - )
fs fy 2

Mu = 513.7 kN.m
fs =
0.003


= 500
3. Description of beam
(500)(0.003) 1. Depth of compression block for a
=
(500304.55)(0.003) balanced condition.
= 304.55

= 0.00193


=

400
= 200000

= 0.002

<

=
0.00193 < 0.002
414
The beam is Over Reinforced. = 200000

= 0.002
PROBLEM 45: 0.003 0.002

= 500
A beam has a width of 300 mm and an effective
c = 300 mm
depth of 500 mm. fc = 28 MPa, fy =414 MPa, Es
= 200,000 MPa
a = c
1. Determine the depth of compression
a = 0.85 (300)
block for a balanced condition.
a = 255 mm
2. Determine the balanced steel area
required.
2. Balanced steel area required.
3. Determine the moment capacity for
C=T
maximum steel area in a balanced
0.85 fc ab = As fy
condition.
0.85 (28) (255) (300) = Asb (414)
Asb = 4398 mm2
Solution:

3. Moment capacity for maximum steel area


in a balanced condition.
As = 0.75 Asb (max. steel area for
balanced condition)
As = 0.75 (4398)
As = 3298.5 mm2
1. Total depth of the beam for a balanced
condition.

= = 0.002 < 0.005


Use = 0.65


=

Mu = As fy (d - 2
)
414
255 = 200000
Mu = 0.65 (3298.5) (414) (500 - 2
)
= 0.0021
Mu = 330.64 kN.m

a = c
255 = 0.85 (c)
c = 300 mm
PROBLEM 46:
The width of a rectangular beam is 300 mm. The
By ratio and proportion:
depth of compression block for a balanced
0.003 0.0021
condition is 255 mm. fc = 28 MPa, fy =414 MPa, 300
= 300

Es = 200,000 MPa. Use 70 mm as steel covering. d = 300 + 210


Unit weight of concrete is 24kN/m3. d = 510 mm
1. Determine the total depth of the beam for
a balanced condition. Total depth = 510 + 70 = 580 mm
2. Determine the total area of reinforcement
for a balanced condition. 2. Area of reinforcement for a balanced
3. Determine the factored support imposed condition.
uniform load that a 6-m. simple span
beam could support for a balanced T=C
condition. Asb fy = 0.85 fc ab
Asb (414) = 0.85 (28) (255) (300)
Solution: Asb = 4398 mm2
3. Factored super imposed uniform load Solution:
that a 6-m. simple span beam could 1. Depth of compression block for a
support for a a balanced condition. balanced condition.


Mu = As fy (d - 2)
255
Mu = 0.90 (4398) (414) (500 - 2
)

Mu = 626.8 kN.m

2
Mu = 8
(6)2
626.8 =
8
=
Wu = 139.29 kN/m

415
= 200000
Wt. of concrete = 0.3 (0.58) (24) = 0.0020750
Wt. of concrete = 4.18 kN/m

0.003 0.0020750
=
Wu = 4.18 (1.2) + Ws 500

c = 295.57 mm
139.29 = 4.18 (1.2) + Ws
Ws = 134.27 kN/m
a = c

PROBLEM 47: a = 0.85 (295.57)

A reinforced concrete beam has a width of 250 a = 251.23 mm

mm and an effective depth of 500 mm. The


compression strength of concrete is 28 MPa and 2. Steel area required for a balanced

the yield strength of steel is fy = 415 MPa. condition.

1. Determine the depth of compression


block for a balanced condition.
2. Determine the steel area required for a
balanced condition.
3. Determine the ultimate moment capacity
to ensure that concrete fails in a ductile
manner.
Check for :
C=T
0.85 fc ab = Asb fy
0.85 (28) (251.53) (250) = Asb (415)
Asb = 3606 mm2 (balanced steel
area)

3. Ultimate moment capacity to ensure that


concrete fails in a ductile manner.

Use As = 0.75 Asb


a = c
As = 0.75 (3606)
188.63 = 0.85 (c)
As = 2704.50 mm2
c = 221.92 mm

0.003
278.08
= 221.92

= 0.00376 < 0.005

(250)
Use = 0.65 + ( 0.002) 3
(250)
= 0.65 + (0.00376 0.002) 3

= 0.80


Mu = As fy (d - 2)
188.63
C=T Mu = 0.80 (2704.05) (415) (500 - 2
)
0.85 fc ab = Asb fy Mu = 364.26 kN.m
0.85 (28) (a) (250) = 2704.50 (415)
a = 188.63 PROBLEM 48:
A rectangular reinforced concrete beam has a

Mu = As fy (d - 2
) width b = 300 mm and an effective depth d = 400
mm. If fc = 28 MPa, fy = 280 MPa, Es = 200,000
MPa.
1. Which of the following gives the nearest 2. Compressive force of concrete:
vaule of the distance of the N.A. from the
top of the beam so that the strain in
concrete = 0.003 will be attained at
the same time with the yield strain of
steel .
2. Which of the following gives the nearest
value of the total compressive force of
concrete.
3. Which of the following gives the nearest
value of the balanced steel ratio.

a = c
Solution:
a = 0.85 (272.73)
1. Distance from N.A. to the top of the
a = 231.82 mm
beam:

C = 0.85 fc ab

= C = 0.85 (28) (231.82) (300)

280 C = 1655194.8 N
= 200000
C = 1655.2 kN
= 0.0014

3. Balanced steel ratio:


0.003 0.0014
= 0.85 600
400
= (600+ )
c = 272.73 mm
0.85 (28)(0.85)600
= 280(600+280)

= 0.0493
= . %

PROBLEM 49:
The beam has a cross section as shown in the
figure. It carries an ultimate moment of 156
kN.m. Using fc = 20.7 MPa, fy = 414 MPa, Es =
200,000 MPa.
T=C
509804 = 0.85 fc Ac
509804 = 0.85 (20.7) Ac
Ac = 28974 mm2
Ac = 100 (a) (2) + 100(a 75)
28974 = 200a + 100a 7500
300a = 36474
a = 121.58 mm
a = c
121.58 = 0.85 (c)
1. Compute the location of the neutral axis c = 143.04 mm
measured from the top of the beam.
2. Compute the number of 20 mm bars 2. Number of 20 mm
needed. A1 = 100 (121.58) (2)
3. Compute the actual strain of the steel A1 = 24316
reinforcements used. A2 = 100 (46.58)
A2 = 4658
Solution:
1. Location of N.A.
A = A1y1 + A2y2
(24316 + 4658) = 24316 (60.79) + 4658 (98.29)
= 66.82

z = 400 66.82 = 333.18 mm

Mu = T z
156 x 106 = 0.90 As (414) (333.18)
As = 1256.62 mm2
Approximate z = 0.85 d
z = 0.85 (400)
4
(20)2 = 1256.62
z = 340
N = 4 bars
Mu = T z
Use 4 20 mm bars
156 x 106 = 0.90 T (340)
T = 509804
3. Actual strain of steel bars
Solution:
0.003 1. Location of neutral axis from the top of
256.96
= 143.04
the beam for a balance condition.
= . (actual strain)

=

414
=
200000

= 0.00207
> steel yields
=

PROBLEM 50:
A symmetrical cross-section of a reinforced
concrete shown has a value of fc = 24.13 MPa, fy
= 482.7 MPa, Es = 200,000 MPa.

482.7
= 200000

= 0.00241

0.003 0.00241

= 681.25

c = 377.77 mm

2. Balanced steel area Asb

1. Which of the following gives the location


of neutral axis from the top of the beam
for a balance condition.
2. Which of the following gives the
balanced steel area Asb.
3. Which of the following gives the max.
area permitted by the code.
a = c 3. Which of the following gives the max.
a = 0.85 (377.77) area permitted by the code.
a = 321.10 mm
Solution:
C = 0.85 fc ab 1. Location of neutral axis from the top of
C = 0.85 (24.13) [375(125) + 125(321.10)] the beam for a balance condition.
C = 1784670 N

T=C
Asb (482.7) = 1784670
Asb = 3697 mm2

3. Max. area permitted by the code


Asmax = 0.75 (3697)
Asmax = 2773 mm2

=

PROBLEM 51: 345
=
The hallow box beam in the figure must carry a 200000

factored moment of 540 kN.m. fc = 28 MPa, fy = = 0.0017

345 MPa, Es = 200,000 MPa.


0.003 0.0017

= 725

c = 462.77 mm

2. Balanced steel area Asb


a = c
a = 0.85 (462.77)
a = 393.35 mm

C = 0.85 fc A

1. Which of the following gives the location A = 125 (393.35)(2) + 250(150)

of neutral axis from the top of the beam A = 135,837.5 mm2

for a balance condition.


2. Which of the following gives the C = 0.85 (28) (135837.5)

balanced steel area Asb. C = 3232933 N


T=C T=C
Asb (345) = 3232933 Asb fy = 0.85 fc Ac
2
Asb = 9371 mm (12)2 (3) (414) = 0.85 (20.7) (255) Ac
4

Ac = 7983.3 mm2
3. Max. area permitted by the code
Asmax = 0.75 (9371)
2
= 7983.3
2
Asmax = 7028.25 mm
15966.7
=

PROBLEM 52:
A triangular beam having a base width of 300 By ration and proportion
300
mm. has a total depth of 600 mm. It is reinforced

= 600
with 3 12 mm bars placed at 70 mm above the = 0.5
bottom of the beam. fc = 20.7 MPa, fy = 414 MPa,
Es = 200,000 MPa. 15966.7
0.5 =

= 178.70
1. Compute the neutral axis of the beam
from the apex of the section.
a = c
2. Compute the ultimate strength capacity
178.70 = 0.85 (c)
of the beam.
c = 210.24 mm
3. What would be the steel area required for
a balanced condition?
2. Ultimate strength capacity
z = 530 2/3 (178.7)
Solution:
z = 410.87
1. Neutral axis of the beam

Mu = T z

Mu = 0.90 4 (12)2 (3) (414) (410.87)

Mu = 51.9 kN.m

3. Steel area required for a balanced


condition.
3
As = 4 Asb

= 3
As = 4 (755)
414
= 200000 As = 566.25 mm2
= 0.00207
0.003 0.00207

= 530 PROBLEM 53:
1.59 0.003c =
A triangular beam has an effective depth of
0.00207
c = 313.61 687.50 mm and a base of 750 mm. The beam
carries an ultimate moment of 197 kN.m. fc =
27.5 MPa, fy = 414 MPa

1. Compute the neutral axis of the beam


2. Compute the value of total compressive
force of concrete.
3. Compute the streel area required.

Solution:
1. Neutral axis of the beam.

a = c
a = 0.85 (313.61)
a = 266.57 mm

T=C
Asb fy = 0.85 fc Ac
266.57 600

= 300
750
x = 133.29
= 750

a=b
Asb fy = 0.85 fc Ac
(133.29)(266.57) C=T
Asb (414) = 0.85 (20.7) 2
0.85 fc 2
= As fy
Asb = 755 mm2
0.85(27.5)()2
2
= (414.7)

As = 0.0282 a2
PROBLEM 54:
Mu = T z The beam has a cross section shown in the figure.
2
197 x 106 = 0.90 As (414.7) (687.50 - 3 a) fc = 20.7 MPa, fy = 414.7 MPa

197 x 106 = 0.90 (0.282) a2 (414.7)


(687.50 0.667a)
18717186.73 = 687.50a2 0.667a3

Solve for a by trial and error


a = 181.8
a = c
181.8 = 0.85 (c)
c = 213.88 mm

2. Compressive force of concrete 1. Compute the minimum steel area



C = 0.85 fc 2 permitted by the NSCP Specs.
0.85 (27.5)(181.8)(181.8) 2. Compute the flexural design strength
= 2
Mn if it is reinforced with minimum steel
C = 386286 N
area.
C = 386.3 kN
3. Compute the maximum area of flexural
steel that can be used in reinforced the
3. Steel area required
section.
C=T
386286 = As fy
Solution:
386286 = As (414.7)
1. Min. steel permitted by the NSCP Specs.
As = 931.5 mm2 1.4
=

1.4
=
414.7

= 0.00338

=
= 0.00338 (125)(2)(437.5)
= .

2. Flexural strength Mn
C=T
0.85 fc (750) a = As fy
0.85 (20.7) (750) a = 369.2 (414.7)
a = 11.60 mm


Mn = 0.90 T ( )
2
11.6
Mn = 0.90 (369.2) (414.7) (437.5 2
)

Mn = 59.5 x 106 N.mm


Mn = 59.5 kN.m
1. Determine the neutral axis of the section
3. Max. steel area that can be used from the top of the beam.
0.85 600 2. Determine the flexural capacity of the
= (600+ )
cross section.
0.85 (20.7)(0.85)600
= 3. Determine the strain in the steel at failure.
414.7(600+414.7)

= 0.0213
Solution:
= 0.75 1. Neutral axis of the beam.
= 0.75 (0.0213)
= 0.016

=
= 0.016 (125)(2)(437.5)
=

PROBLEM 55:
C=T
Each leg of the cross section is reinforced with a
0.85 fc Ac = As fy
28 mm bar.

0.85 (24.8) (500) a = 4 (28)2 (2) (414.7)
a = 48.45 mm
a = c
48.45 = 0.85 (c)
c = 57 mm
2. Flexural capacity Mn

Mn = 0.90 T ( 2 )
48.45
Mn = 0.90 (28)2 (2) (414.7) (375 )
4 2

Mn = 161.2 x 106 N.mm


Mn = 161.2 kN.m

3. Strain in steel at failure:


= 1963.5 2
0.003
=
318 57

= .
=
1963.5
= 300(380)
PROBLEM 56: CE BOARD MAY 2010
A 12 m simply supported beam is provided by an = 0.017

additional support at midspan. The beam has a


width of b = 300 mm and a total depth h = 450 If only tension bars are needed:

mm. It is reinforced with 4 25 mm at the = 0.75( )


tension side and 2 25 mm at the compression = 0.023 > 0.017
side with 70 mm cover to centroid of
reinforcement. fc = 30 MPa, fy = 415 MPa. Use Therefore, the beam needs only tension

0.75 = 0.023 bars a specified in the problem.

1. Determine the depth of the rectangular C=T

stress block. 0.85 fc ab = As fy



2. Determine the nominal bending moment, 0.85 (30) (a)(300) = (25)2 (4)(415)
4
Mn. a = 106.52 mm
3. Determine the total factored uniform load
including the beam weight considering 2. Nominal bending moment:
moment capacity reduction of 0.90. Mn = As fy ( 2 )

Solution:
106.52
Mn = 4 (25)2 (4)(415) (380 )
1. Depth of the rectangular stress block: 2

Mn = 266.2 x 106 N.mm


Mn = 266.2 kN.m
Check if compression bars is needed.

= (25)2 (4)
4
3. Total factored uniform load including PROBLEM 57:
beam weight: A rectangular beam has a width of 300 mm and
an effective depth of 537.50 mm to the centroid
of tension steel bars. Tension reinforcement
consists of 6 28 mm in two rows, compression
reinforcement consists of 2 22 mm . fc = 27.6
MPa, fy = 414.7 MPa. Assume steel covering is
60 mm for compression bars.

5 4
=
384 1. Compute the depth of compression
3
= 48
block.
5 4 3 2. Compute the factored moment capacity
384
= 48
of beam.
5
= 8 3. Compute the maximum total tension steel
2R + P = wL allowed by specifications.
5
2R = wL - 8
3 Solution:
2R =
8
1. Depth of compression block.
3
R = 16


= ( ) ( ) ( )
2 2 4
32 2
= 16(2)
8

2
= 32

Mu = 0.90 Mn
Mu = 0.90 (266.2)

Mu = 239.58 kN.m = (28)2 (6)
4
2
239.58 = = 3695 2
32
(12)2
239.58 =
32
= 4 (22)2 (2)
w = 53.24 kN/m
= 760 2


=
=
3695 As1 = As As2
300(537.50)
As2 = As when compression bars will yield
= 0.0229

As1 = As As
=
760
=
300(537.50)
0.85 fc ab = (As As) fy

= 0.00471 0.85 (27.6) a (300) = (3695 760) 414.7


a = 172.94 mm

Check if compression bar are really needed:


0.85 600 2. Factored moment capacity of the beam.
=
(600+ )
a = c
0.85 (27.6)(0.85)600
= 414.7(600+414.7)
172.94 = 0.85 (c)

= 0.0284 c = 203.46 mm
1 0.003
When, 346.6
= 203.4
> compression bars are needed 1 = 0.00511 > 0.005
= 0.75
0.0229 > 0.75(0.0284) Use capacity reduction factor: = 0.90
0.0229 > 0.0213
Check: if compression bars will yield
0.85 600
>
(600 )

= 0.0229 0.00471
= 0.0182


0.85 (27.6)(0.85)(60)600 M1 = As1 fy ( 2 )
=
537.5(414.7)(600414.7)
As2 = As
= 0.0173
As1 = As - As2
As1 = As As
0.0182 > 0.0173 (compression bars will yield)

M1 = (As As) fy ( 2 )
172.94
Check: M1 = 0.90 (3695 760) 414.7 (537.50 2
)
< M1 = 494.07 x 106 N.mm
0.0182 < 0.0213

M2 = As2 fy ( )
C1 = T1 M2 = As fy ( )
0.85 fc ab = As1 fy
M2 = 0.90 (760) (414.7) (537.5 60) 630 = 1.8 b
M2 = 135.45 x 106 N.mm b = 350 mm

Mu = M 1 + M 2 2. Reinforcement for compression


Mu = 494.07 + 135.45 0.85 600
=
(600+ )
Mu = 629.52 kN.m
0.85 (27.58)(0.85)600
= 413.4(600+413.4)

3. Maximum total tension steel allowed by = 0.0285


specifications.
0.85 fc ab = As1 fy

Max. As = +
Max. As = 0.75 +
Max. As = (0.75 + )
Max. As = 300(537.5)(0.0213 + 0.00471)
Max. As = >

PROBLEM 58:
A doubly reinforced concrete beam has a max. Assume
effective depth of 630 mm and is subjected to a 0.85 (0.003)
1 = (0.008)
total factored moment of 1062 kN.m including its
0.85 (27.58)(0.85)(0.003)
own weight. fc = 27.58 MPa, fy = 413.4 MPa. Use 1 =
413.4 (0.008)

62.5 mm steel covering. 1 = 0.018

1. Determine the width of the beam As1 = As As2


2. Determine the reinforcement for As2 = As if compression steel will yield
compression As1 = 1 bd
3. Determine the total reinforcement for As1 = 0.018 (350) (630)
tension As1 = 3969 mm2

Solution:
M1 = As1 fy ( 2 )
1. Width of the beam
199.97
Approximate proportion of b and d M1 = 0.90 (3969) 413.4 (630 )
2

d = 1.5 to 2 b M1 = 782.7 kN.mm


Try d = 1.80 b Mu = M 1 + M 2
1062 = 494.07 + M2 Use As = 5292 mm2
M2 = 279.3 kN.m
PROBLEM 59:
Check whether compression bars will yield: A rectangular beam has a width of 300 mm and
0.85 600 an effective depth to the centroid of the tension
>
(600 )
reinforcement of 600 mm. The tension
As = As1 + As2 2
reinforcement has an area of 4762 mm and the
As2 = As
area of compression reinforcement placed 62.50
As = As1 + As
mm from the compression face to the beam is
bd = 1 bd + bd
987.5 mm2. fc = 34.56 MPa, fy = 414.6 MPa.

= 1
Balanced steel ratio is 0.034. Assume that steel
0.85 (27.58)(0.85)(62.5)(600)
0.018 > 630 (413.4)(600413.4) yields.
0.018 > 0.0154
1. Determine the depth of compression
Therefore, compression bars will yield. block
fs = fy 2. Determine the design strength using 0.90
As fy = As2 fy as the reduction factor
As = As2 (ok as assumed) 3. Determine the concentrated live loads at
M2 = As fy ( ) the midspan in addition to a dead load of
279.3 x 106 mm = 0.90 As (413.4) (630 62.5) 20 kN/m including the weight of the
As= 1323 mm2 beam if it has a span of 6 m.

3. Total reinforcement for tension. Solution:


As = As1 + As2 1. Depth of compression block
As = 3969 + 1323
As = 5292 mm2

=

1323
= 350(630)

= 0.006
Max. As = bd (0.75 + )
Max. As = 350(630) [0.75(0.0285) +
0.00669)]
Max. As = 6036 mm2 > 5292 mm2 ok
0.85 600
= (600+ )

0.05 ( 28)
= 0.85 7
0.05 (34.5628)
= 0.85 7

= 0.803
0.85 (34.56)(0.803)(600)
= 414.6(600+414.6)

= 0.034

4762 Since, compression bars will yield:


= = 300 (600)
fs = fy
= 0.0264
T = C1 + C2
987.5
=
= 300 (600) As fy = 0.85 fc ab + As fy
= 0.0054 ( )
=
0.85
(4762987.5)414.6
Check the beam first as a singly reinforced beam = 0.85 (34.56)(300)

to see if the compression bars can be disregarded. = .


= 0.75
= 0.75 (0.034) 2. Design strength using 0.90 as the
= 0.0255 reduction factor.
= 0.0264 > 0.0255

Therefore, the beam must be analyzed as doubly


reinforced beam. Check if the bars in
compression will really yield, by computing the
steel ratio that will ensure yielding of the
compression bar at failure.
= 0.0264
0.85 600 C1 = T1
> (600 )
0.85 fc ab = As1 fy
0.85 (34.56)(0.817)(62.5)(600)
0.0264 0.0054 > 414.6 (600)(600414.6)
0.85 (34.56) (177.57) (300) = As1 (414.6)
As1 = 3774.50 mm2
0.021 > 0.0195 (therefore, compression
bars will yield)
As = As1 + As2
As2 = As As1 PROBLEM 60:
As2 = 4762 3774.50 A reinforced concrete beam has a width of 375
As2 = 987.50 mm2 ok mm and a total depth of 775 mm. Steel covering
for both compression and tension bars is 75 mm.
Mu = (M1 + M2) Area of compression bars is 1290 mm2 while that

M1 = T1 ( 2 )
of the tension bars it is 6529 mm2. fc = 27.6 MPa,


fy = 414.6 MPa.
M1 = As1 fy ( 2 )
177.51
M1 = 3774.5 (414.6) (600 ) 1. Determine the depth of compression
2

M1 = 800 x 106 N.mm block.


2. Determine the capacity reduction factor
M2 = As2 fy ( ) for moment.

M2 = 987.50 (414.6) (600 62.5) 3. Determine the ultimate moment capacity


M2 = 220 x 106 N.mm of the beam.

Mu = (M1 + M2) Solution:


Mu = 0.90 (800 + 220) 1. Depth of compression block.

Mu = 918 kN.m

3. Concentrated live loads it could support


at its midspan.

0.003
=
75
0.003(75)
=

2 =
Mu = (1.6) 4
+ 8
(1.2)
0.003(75)(200000)
=
(6) 20(6)2
918 = (1.6) 4
+ 8 (1.2)
600(75)
=
P = 337.5 kN
T = C1 + C2
As fy = 0.85 fc ab + As fs
6529 (414.6) = 0.85(27.6) (0.85) c (375)
1290 (600)(75)
+

1932923.4 c = 7477.88c2 58050000 = 0


c2 258.49 c 7762.90 = 0
c = 285.66 mm

600 (75) Note:


=

600 (285.6675) When < 0.005
=
285.66 (250)
Value of = 0.65 + ( 0.002)
3
= 442.47 > 414.6
(250)
= 0.65 + (0.00435 0.002) 3

Compression steel yield: = 0.846


Use fs = fy = 414.6 MPa
a = c 3. Ultimate moment capacity of the beam.
a = 0.85 (285.66)
c = 242.81 mm

2. Capacity reduction factor for moment.


0.003

=
0.003()
=

=
C2 = T2
0.003(700285.66)
285.66 As fy = As2 fy
= As = As2
0.00435 < 0.005 As2 = 1290 mm2

As1 = As As2
As1 = 6529 1290
As1 = 5239 mm2
Solution:
= [1 ( 2 ) + 2 ( )]

1. Minimum tensile steel ratio that will
= [1 ( 2 ) + 2 ( )]
ensure yielding of the compression steel
242.81
= 0.846 [5239(414.6) (700 2
)+ at failure.
0.85 600
1290(414.6)(700 75)] = +
(600+ )

= 1346 106 . As = = (28)2 (2) = 1232 2
4
= . 0.85 600
= (600+ )
+

0.85 (20.7)(0.85)(62.5)600 1232


PROBLEM 61: = +
414.6(537.5) (600+414.6) 350(537.5)
A rectangular concrete beam has a width of 350
= 0.020
mm and a total depth of 675 mm. It is reinforced
for tension at the bottom with 4 36 mm bars
Check:
at an effective depth of 537.5 mm and two 28 mm
=

bars at the top placed at 62.5 mm from the top

of the beam. fc = 20.7 MPa, fy = 414.6 MPa. As = = (36)2 (4) = 4071.5 2
4
4071.5
= 350(537.5)
1. Determine the minimum tensile steel
= 0.0216 > 0.020
ratio that will ensure yielding of the
= .
compression steel at failure.
2. Determine the total compressive force of
2. Total compressive force of concrete.
concrete.
3. Determine the design moment capacity
of the beam.


As = = (36)2 (4) = 4071.5 2
4

As = 4 = (28)2 (2) = 1232 2
4071.5
=
350(537.5)

= 0.0216
1232
= 350(537.5)

= 0.0065
C1 = 0.85 (20.7) (188) (350)
When: C1 = 1157751 N
0.85 600
> (600 )
(compression bars will
C2 = As fs
yield)
C2 = 1232 (414.6)
= 0.0216 0.0065
C2 = 510787 N
= 0.0151
0.85 (20.7)(0.85)(62.5)600
0.0151 >
414.6(537.5)(600414.6) M1 = C1 ( 2 )
0.0151 > 0.0136 188
M1 = 1157751 (537.5 )
2

M1 = 513.46 x 106 N.mm


Therefore, compression steel yields.
M2 = C2 ( )
fs = fy = 414.6 MPa
M2 = 510787 (537.5 62.5)
C1 = 0.85 fc ab
M2 = 242.62 x 106 N.mm
a = c
a = 0.85 (221.18)
Mu = (M1 + M2)
a = 188 mm
Mu = 0.90 (513.46 + 242.62)
C1 = 0.85 (20.7) (188) (350)
Mu = 635.11 kN.m
C1 = 1157751 N
C1 = 1157.75 kN
PROBLEM 62: CE Board Nov. 2010,
Nov.2012
3. Design moment capacity of the beam.
A simply supported beam is reinforced with 4
0.003 28 mm at the bottom and 2 28 mm at the top
316.32
= 221.18
of the beam. Steel covering to centroid of
= 0.00429 <
reinforcement is 70 mm at the top and bottom of
0.005
the beam. The beam has a total depth of 400 mm
and a width of 300 mm. fc = 30 MPa, fy = 415
Use = 0.65 + (
MPa. Balanced steel ratio = 0.031.
(250)
0.002)
3
(250) 1. Determine the depth of compression
= 0.65 + (0.00429 0.002) 3
block.
= 0.84
2. Determine the design strength using 0.90
as the reduction factor.
Mu = (M1 + M2)
C1 = 0.85 fc ab
3. Determine the live load at the mid-span
in addition to a DL = 20 kN/m including Therefore, compression bars will not yield.
the weight of the beam if it has a span of fs fy
6 m. 0.003


= 70
0.003(70)
=
Solution:
=
1. Depth of compression block:

As = = (28)2 (4) = 2463 2
4

As = = (28)2 (2) = 1231.5 2
4


=
2463
=
350(330)

= 0.0248 T = C1 + C2
As fy = 0.85 fc ab + As fs
= 0.75 2463(415) = (30) (a) (300) + 1231.5
0.003(70)(200000)
= 0.75 (0.031)

= 0.023 738900(70)
1022145 = 7650a +
738900(70)
> 1022145 = 7650 (0.85c) +

Therefore, reinforcement for compression is 157.19 c = c2 + 113.63 (c 70)


needed. To ensure that compression bars will c2 + 43.56 c -7654.10 = 0
43.56 183.61
yield. = 2
0.85 600
> (600 )
c = 113.59
0.85 (30)(0.85)(70)600
>
415(330)(600415)
a = c
> 0.036 a = 0.85 (113.59)

= a = 96.55 mm
1231.5
= 300(330)

= 0.0124
> 0.036
0.0248 0.0124 < 0.036
0.0124 < 0.036
2. Design strength using 0.90 as reduction Mu = (M1 + M2)
factor. Mu = 0.90 (208 + 73.7)
Mu = 253.53 kN.m

3. Concentrated live load at mid-span:


2
Mu = (1.6) 4
+ 8
(1.2)
(6) 20(6)2
253.53 = (1.6) 4
+ 8
(1.2)

P = 60.64 kN

C1 = 0.85 fc ab
C1 = 0.85 (30) (96.55) (300) PROBLEM 63:
C1 = 738607.50 N A reinforced concrete beam has a width of 350
mm and an effective depth of 562.5 mm. It is

C2 = As fs reinforced for tension at the bottom of the section

= having an area of 4896 mm2 and for compression


0.003(70) at the top of the beam 62.5 mm below the extreme
=
compression fibers of the beam, having an area of
0.003(113.5970)
= 113.59 1530 mm2. fc = 34.6 MPa, fy = 414.7 MPa.
= 0.0011512
= 0.0011512 (200000) 1. Determine the depth of compression
= 230.25 < 415 block.
2. Determine the ultimate moment capacity.
C2 = As fs 3. Determine the maximum total tension
C2 = 1231.5 (230.25) area that could be used in this section.
C2 = 283553
Solution:
1. Depth of compression block.
M1 = C1 ( )
2

96.55 =
M1 = 738607.5 (330 2
)
4896
M1 = 208 x 106 N.mm = 350(562.5)

M2 = C2 ( ) = 0.0249
M2 = 283553 (330 70)
M2 = 73.7 x 106 N.mm
=
1530
= 350(562.5)
= 0.0078 C1 + C2 = T1 + T2
C1 + C2 = T + As fy
= 0.0249 0.0078 8234.8 c + 918000
(62.5)
= 4896 (414.7)

= 0.0171 c2 + 111.48 (c 62.5) = 246.56 c
c2 135.08 c 6967.5 = 0
0.85 600
When, > (600 )
c = 174.91 mm

(compression bar did not yield)

a = c
a = 0.85 (174.91)
a = 139.93 mm

2. Ultimate moment capacity.


C2 = As fs
0.003 600(174.9162.5)


= = 174.91

=
0.003() = 385.60

91800(174.9162.5)
= 2 = 174.91

2 = 589974
0.003( ) 2 = 589.97
=
(200,000)
600(62.5)
=
C1 = 8234.8 c
C1 = 8234.8 (174.91)
C1 = 0.85 fc ab C1 = 1440349 N
C1 = 0.85 fc cb
C1 = 0.85 (34.6) (0.8) c (350)
M1 = C1 ( 2 )
C1 = 8234.8 c
139.93
M1 = 1440349 (562.5 2
)

C2 = As fs M1 = 709.42 x 106 N.mm


1530 (600)(62.5) M2 = C2 ( )
C2 =
(62.5)
M2 = 589974 (562.5 62.5)
C2 = 918000
M2 = 294.984 x 106 N.mm

Mu = (M1 + M2)
Mu = 0.90 (709.42 + 294.98)
=
Mu = 903.96 kN.m 4744
= 375(500)

3. Maximum total tension area that could be = 0.0253

used in this section.




Max. As = bd ( 0.75 +

) =
1968
Max. As = 350(562.5) [0.75(0.03346) + = 375(500)
0.0078 (385.60)
] = 0.010496
414.7

Max. As = 6369 mm2 = 0.75

PROBLEM 64: 0.85 600


=
(600+ )
A rectangular concrete beam has a width of 375
0.85 (27.6)(0.85)(600)
=
mm and an effective depth of 500 mm. 414.6(600+414.6)

compression bars has an area of 1968 mm2 = 0.02844


located at 100 mm from the compression face of
the beam. The tension bars have an area of 4744 = 0.75(0.02844)
mm2. fc = 27.6 MPa, fy = 414.6 MPa. = 0.02133

1. Determine the depth of the compression


0.0253 > 0.02133
block.
Therefore, compression bars are really needed.
2. Determine the max. steel ratio.
3. Determine the ultimate moment capacity
of the beam. Check if compression bars will yield or not.
0.85 600
> (600 )
compression bars will
Solution: not yield
1. Depth of the compression block. 0.85(27.6)(0.85)(100)600
> 414.6 (500)(600414.6)

0.0253 0.010496 < 0.03113


0.0148 < 0.03113 compression bars will
not yield
fs fy
600( )
=

C1 + C2 = T1 + T2
As = As1 + As2 3. Ultimate moment capacity of the beam.
T1 + T2 = T
As1 fy + As2 fy = T
(As1 + As2) fy = T
T = As fy

C1 + C2 = T
0.85 fc ab + As fs = As fy As fs = As2 fy

1968 (600)(100)
1968 (282.15) = As2 (414.6)
0.85 (27.6) (0.85) c (375) + =
As2 = 1339 mm2
4744 (414.6)
c2 105.12 c 15790.57 = 0
As = As1 + As2
c = 188.77 As = 4744 + 1339
a = c As = 3405 mm2
a = 0.85 (188.77)
a = 160.45 mm
M1 = As1 fy ( )
2
160.45
M1 = 3405 (414.6) (500 )
2. Max. steel ratio. 2

M1 = 704.44 x 106 N.mm


= 0.75 +

600( )
=
M2 = As2 fy ( )
600(188.77100) M2 = 1339 (414.6) (562.5 62.5)
= 188.77
M2 = 222.06 x 106 N.mm
= 282.15

Mu = (M1 + M2)
282.15
= 0.02844 + 0.010496
414.6

= 0.02847 Check if we could use = 0.90

1 0.003
0.0253 < 002847
311.23
= 188.77
< ok 1 = 0.005 ok

Use = 0.90 As = 4 = (32)2 (6) = 4825 2
Mu = 0.90 (704.44 + 222.06)
=
Mu = 833.85 kN.m
4825
=
300(600)

PROBLEM 65: = 0.0268


A rectangular beam has a width of 300 mm and
an effective depth to the centroid of the tension = 0.75
reinforcement of 600 mm. The tension 0.85 600
= (600+ )
reinforcement consists of 6 32 mm bars placed
0.05 (28)
in two rows. Compression reinforcement = 0.85 7
consisting of two 25 mm bars is placed 62.5 mm 0.05 (34.628)
= 0.85
7
from the compression face of the beam. fc = 34.6
= 0.80
MPa, fy = 414.7 MPa. 0.85 (34.56)(0.80)(600)
= 414.7(600+414.7)

1. Determine the depth of compression = 0.0335


block.
2. Determine the maximum steel ratio. = 0.75(0.0335)
3. Determine the design moment capacity = 0.0252
of the beam.
Solution: Compression bars is needed.
1. Depth of compression block. >
Check whether compression bars is really 0.0268 > 0.0252
needed. Check if compression bars will yield or not.
0.85 600
> (600 )


=

(25)2 (2)
4
= 200(600)

= 0.00545
0.85(34.6)(0.817)(62.5)600
> 414.7 (500)(600414.7)

0.0268 0.00545 > 0.01954


0.02135 > 0.01954
fs = fy (steel compression yields)

As = 4 = (25)2 (2) = 981.75 2 a = c
180.64 = 0.85 (c)
c = 221.10 mm
T1 + T2 = C1 + C2
0.003
=
T = C1 + C2 378.30 221.10

= 0.0051 >
As fy = 0.85 fc ab + As fy
4825 (414.7) = 0.85 (34.6) (a) (300) + 981.75
fs = fy
Use = 0.90


Mn = C1 ( 2 ) + C2 ( )

Mn = 0.85 fc ab ( 2 ) + As fy ( )

C1 = 0.85 fc ab
C1 = 0.85 (34.6) (180.64) (300)
C1 = 1593787 N
(414.7)
a = 180.64 mm C2 = As fy
C2 = 981.75 (414.7)
2. Maximum steel ratio C2 = 407132 N
= 0.75 +
= 0.75 (0.0335) + (0.00545)
Mn = C1 ( ) + C2 ( )
2
= .
180.64
Mn = 1593787 (600 ) + 407132 (600
2

3. Design moment capacity of the beam. 62.5)


Mn = 1031 x 106 N.mm

Mu = Mn
Mu = 0.90 (1031)
Mu = 927.9 kN.m
PROBLEM 66:
A rectangular beam reinforced for both tension T = C1 + C2
and compression bars has an area of 1250 mm 2
As fy = 0.85 fc ab + As fs
and 4032 mm2 for tension bars. The tension bars 4032 (414.6) = 0.85(20.7) (0.85) c (350)
are placed at a distance of 75 mm from the bottom 1250 (600)(62.5)
+

of the beam while the compression bars are
1671667.2 c = 5234.5c2 750000 c - 46875000
placed 62.5 mm from the top of the beam. fc =
c2 176.08 c 8955 = 0
20.7 MPa, fy = 414.60 MPa. Width of beam is 350
c = 217.29 mm
mm with a total depth of 675 mm.

0.003(62.5)
=
1. Determine the depth of compression
0.003(217.2962.5)
block. =
217.29
2. Determine the ultimate moment of = 0.00214
capacity of the beam.
3. Determine the safe live concentrated load
=

that the beam could support at its mid-
414.6
span if it has a span of 6-m. Assume = 200000

weight of concrete to be 23.5 kN/m. = 0.00207


>
Solution: >
1. Depth of the compression block. a = c
a = 0.85 (217.29)
a = 184.70 mm

2. Ultimate moment capacity of the beam

0.003

=
0.003

= 62.5 0.003()
=
0.003(62.5)
= 0.003(600217.29)
= 217.29

=
=
0.003(62.5)(200000) 0.00529 > 0.005
=
600(62.5)
=
Note: 184.70
= 0.90 [2782(414.6) (600 2
)+
When < 0.005
1250(414.6)(600 62.5)]
(250)
Use = 0.65 + ( 0.002) 3
= 777.7 106 .
= . .

3. Safe concentrated live load at mid span


of the beam:
2
Mu = 4
+ 8
()2
Mu = (1.7) 4
+ 8
(1.4)

WDL = 23.5 (0.35) (0.675)


WDL = 5.55 kN/m
Use = 0.90
()2
Mu = (1.7) + (1.4)
4 8
(6) 5.55(6)2
777.7 = (1.7) + (1.4)
4 8

P = 291.27 kN

As fy = As2 fy
As = As2
As = As1 + As2
As1 = As As2
As1 = As As
As1 = 4032 1250
As1 = 2782 mm2


= [1 ( 2 ) + 2 ( )]

= 0.90 [1 ( 2 ) + 2 ( )]

Potrebbero piacerti anche